You are on page 1of 61

GMAT VERBAL REVIEW

SENTENCE CORRECTION
SENTENCE STRUCTURE

PHRASE A clause is CLAUSE


a group of related SENTENCE
words that must have a subject
Group of related words which
and a verb. Clauses may be Must have a subject
does not express a complete
classified into two categories: Must have a verb
thought, and where the subject,
Must make common-sense/form
verb or both, are missing
An independent clause is one a complete thought
which contains a subject and a
Prepositional phrase: at the
verb, and expresses a complete COMMON ERRORS
beach, near the stove, at this
thought. For example, Jane took Common splice
time
the train This occurs when two
Verbal Phase: building a large
A dependent clause is one independent clauses in a
dam, to fly in a plane, signed
which contains a subject and a sentence are combined using a
the legal document
verb, but does not express a comma instead of using a semi-
complete thought, and cannot colon or an appropriate
Examples:
stand for itself. For example, conjunction
- The sun rises in the east
after Jane took the train. The
- Humpty Dumpty sat on a wall
clause raises a question what Incorrect: Tom read the novel, his
- She wore a hat with blue
happened after Jane took the friend saw the movie
trimming
train? Correct: Tom read the novel, but
- The accident on the bridge
his friend saw the movie
was not serious
Example: Barry ran, as he wanted Correct: Tom read the novel; his
- The girl with red hair is an
to reach on time friend saw the movie
artist
Independent clause: Barry ran
Dependent clause: as he
wanted to reach on time
PARTS OF SPEECH
Part of Definition Example
speech
Noun Concrete nouns: person, animal, thing, place Concrete nouns: teacher, airplane,
Abstract nouns: You cannot see them, hear them, smell them, taste dog
them, or feel them (physically) Abstract nouns: curiosity, love,
happiness
Pronoun Word that takes the place of a noun
Example: I, you, he, she, it, we, them, me, you, him, her, it, us,
them, mine, yours, his, hers, its, ours, theirs, this, that, which, who
Verb Verbs express action something that a person, animal, force of A three-mile run seems like a
nature, or thing can DO. For instance, sneezing, laughing, winking marathon during a hot, humid July
afternoon
State of being linking verbs connect the subject of a verb to
additional information about the subject. Some examples include, any During bad storms, trailer parks are
form of the verb be am, were, has been, are being, might have been often magnets for tornadoes
become, and seem
Adjective A word that modifies a NOUN or a PRONOUN by describing, Smith is a famous explorer
identifying, or quantifying words Sidney Sheldon has written thirty
- What kind? novels
- Which one?
- How much?
Adverb A word that modifies everything but nouns and pronouns modifies Well use the new software program
ADJECTIVES, VERBS AND OTHER ADVERBS. Most adverbs end in ly. tomorrow

A word is an adverb if it answers He climbed the ladder slowly


- How?
- When?
- Where?
- Why?
Conjunction Connecting words: and, but, or, nor, for, yet, so, although, because, Jack and Jill went up the hill
since, unless
Preposition Links nouns, phrases and pronouns to other words in the sentence The book is on the table
THREE KEY CONCEPTS IN SENTENCE CORRECTION

CORRECTNESS Subject verb agreement; Verb tense; Voice & mood; Pronouns; Modifiers; Parallelism; Comparisons; Idioms

Avoid redundancy - if a word can be removed without altering the meaning of the sentence, it is redundant
and should be eliminated
CONCISION Example:
Past experience reveals that cancer patients rarely ever exhibit the exact same symptoms
Refers to In the example above, the word past is implicit in experience and can be eliminated. Similarly the words
ever and exact are only used for emphasis (a practice the GMAT does not condone)
brevity is the Correct: Experience reveals that cancer patients rarely exhibit the same symptoms
sentence
A common redundancy trap is the use of words with the same meaning
written as Example:
economically as The value of the stock rose by a 10% increase
possible? Since rose and increase both imply growth, only one is needed to convey the correct meaning

Being almost always signals redundancy and should be avoided wherever possible

CLARITY
If an answer choice alters the original meaning (or intent) of the sentence, it is incorrect. Most instances of
altered meaning fall into four broad categories:
Refers to 1. Word placement
intelligibility 2. Known vs. Unknown
i. The original sentence is certain about an outcome but the answer choices indicate uncertainty (or vice
is the meaning versa)
of the sentence ii. The original sentence discusses a hypothetical situation but the answer choices present it as an
actual situation (or vice versa)
obvious and 3. Multiple meanings
unambiguous? 4. Such as vs. Like
THREE KEY CONCEPTS IN SENTENCE CORRECTION CLARITY
CONTINUED
Category Description Example
Word Placement Be aware of words that move from All the children are covered with mud
one position to the other; the The children are all covered with mud
placement of a single word can alter
the meaning of the sentence In these sentences, the placement of all shifts the focus
from the number of children covered with mud to the extent
to which the children are covered by it
Known vs. The original sentence is certain about Certain: The court ruled that the plaintiff must pay full
Unknown an outcome but the answer choices damages
indicate uncertainty (or vice versa)
Uncertain: The court ruled that the plaintiff should pay full
damages
The original sentence discusses a Actual: If Newton and Liebniz met, they discussed
hypothetical situation but the mathematics
answer choices present it as an actual
situation (or vice versa) Hypothetical: If Newton and Liebniz met, they would
discuss mathematics
Multiple Some words can be read in more than Sentence # 1: The light fabric makes the shirt easy to fold
meanings one way, altering the meaning of the Sentence # 2: The shirt is easy to fold, and very light
sentence according to the given
interpretation The first sentence makes it clear that in this context the shirt
is not heavy. However, in the second sentence, light could
mean either pale or not heavy. The context does not
establish the meaning clearly
Such as vs. Such as is used to indicate examples; Animals such as zebras and lions live on the Serengeti Plain
Like Like is used to indicate similarity Animals like zebras and lions live on the Serengeti Plain

The first sentence indicates that lions and zebras are
specific types of animals that live on the Serengeti Plain.
The second sentence indicates that animals similar to
lions and zebras live on the Serengeti Plain, but it is not
clear whether lions and zebras themselves actually do
SUBJECT VERB AGREEMENT
The basic rule states that a singular subject takes a singular verb, while a plural subject takes a plural verb

First determine whether the subject in the sentence or the clause is singular or plural

To find the subject, always find the verb first, and then ask who or what performed the verb
It is important to note that verbs do not form their plurals by adding an s as nouns do. In order to determine
which verb is singular and which one is plural, think of which verb you would use with he or she and which verb you
would use with they
If a verb follows to, it is called an infinitive phrase and is not the main verb. You will find the main verb either before or
after the infinitive phrase
Example: The efforts to get her elected succeeded

COLLECTIVE NOUNS ARE GENERALLY


ELIMINATE THE MIDDLEMAN SINGULAR
The intervening phase between the subject and the verb A collective noun is a noun that looks singular (it usually
should be eliminated so that the true subject becomes clear does not end with an s) but refers to a group of people
of and for are two common middlemen
Prepositional phrases Collective nouns are singular when they act in a
collective fashion or represent a group. Examples
Examples: include team, family, majority, administration, army,
The houses of that rich man (contain/contains) very audience, class, crowd, faculty, audience, and committee
expensive furniture
The subject houses is plural, and hence requires the Collective nouns are plural when the members of
plural verb form contain the collective body act as individuals
The team are always fighting amongst themselves
The discovery of new lands (was/were) important to the While 'team' is often used as a singular collective noun,
expansion of the British empire in this case, the sentence describes the fighting that
The building of tall skyscrapers (has/have) increased in occurs between the individual members of the team.
the past few years "Team" therefore refers to several individual members,
The actions of my friend (is/are) not very wise and requires a plural verb, "are," as a result
The book I bought for my students (tell/tells) the story of
a Russian immigrants experience in the United States
SUBJECT VERB AGREEMENT

INFINITE PRONOUNS ARE SINGULAR


AND vs. ADDITIVES
An infinite pronoun is one which is not specific about the thing to
The word and can unite two or more singular subjects, which it refers
forming a compound plural subject
John and his friends ARE going to the beach All pronouns which end in body, one, or thing fall under this
category:
When the two singular noun subjects connected by and refer - Anyone, anything, anybody
to the same person or thing, the singular verb is used - Everyone, everybody, everything
Bread and butter is his staple diet - Each, every
The recently appointed manager and coach of the team has - Whatever, whoever, no one, nobody, nothing
emphasized the need for fitness - Either and neither (may require a plural verb form when
paired with or/nor)
Additive phrases such as: along with, in addition to, as well as, Neither of the rosebushes is as pretty as last year
accompanied by, together with, including, do not form Either of us is capable of doing the work
compound plural subjects Neither John nor his friends are going to the beach
Correct: John, along with his friends, IS going to the beach
Incorrect: Frank, accompanied by his students, were at the There are 5 infinite pronouns which can be either singular or plural
studio depending on the context of the sentence: Some, Any, None, All,
Correct: Frank, accompanied by his students, was at the studio Most (SANAM)

EACH/EVERY GENERALLY SINGULAR


SENSATIONS
VERB FORM IN EITHER/OR , NEITHER/NOR
DEPENDS ON THE SUBJECT NEAREST TO When each/every is the subject, the verb form is singular
THE VERB Each of these shirts IS pretty

In these sentences, there are two subjects. If one subject is When the subject is preceded by each/every, the verb form is
singular, and the other is plural, find the subject nearest to the singular
Every dog and cat HAS paws
verb, and ensure that the verb agrees in number with this subject

Neither Joe nor his friends ARE going to the beach When each/every follows a subject, it has no bearing on the
Neither his friends nor Joe IS going to the beach verb form
They each ARE great tennis players
The apartments each HAVE their own private entrance
SUBJECT VERB AGREEMENT
NUMERICAL The phrase, the number of always takes a The number of students in this class IS quite large
WORDS AND singular verb form
PHRASES A number of students in this class ARE hardworking
The phrase, a number of always takes a plural
verb form TWO out of every three dog owners in the U.S. also
OWN a cat
Numbers greater than 1 are plural
PORTIONS With words that indicate portionspercent, fraction, Half of the dresses ARE dirty
part, majority, some, all, none, remainder, and so forth
look at the noun in your of phrase (object of the Half of the sugar IS over
preposition) to determine whether to use a singular or
plural verb Two-thirds of the work IS complete

If one means the many individual parts of the totality, The majority of Congress has voted for the bill
then use a plural verb form
Example:
The majority of the students in the class ARE hard
working

If one means the totality itself, then use a singular verb
form
Example:
The student majority IS opposed to the death penalty

ONE OF The phrases one of and one of the take a singular verb One of you is telling the truth
One of the editors wants a rewrite
Plural - The phrases one of those who and one of the The comma splice is one of those errors that always
things/factors slip past me
Singular - When the word the only precedes these One of the things that drive me nuts is subject-verb
phrases agreement
Meg is the only one who knows how to paddle a
canoe
Ted is not the only one of my nephews who has a
vivid imagination

MISCELLANEOUS
Both... and is the ONLY pair that always results in a plural subject
Gerunds (-ing form of the verb and functioning as nouns) are singular subjects
When in doubt, think singular
VERB TENSE
VERB TENSE indicates when an action takes place. The basic tenses are present, past and future

Unless actions do not take place at the same time, keep all verb tenses in a given sentence the same
Example
She walked to school in the morning and ran home in the afternoon
She walks to school in the morning and runs home in the afternoon
She will walk to school in the morning and run home in the afternoon (Note: run is understood as will run)

Typically, ing forms are sometimes used as junk answers on the GMAT; there will often be a better alternative
Tip: Pick one event as the base action determine when the other actions occurred in relation to that event determine the
proper verb form
Will indicates future tense; would indicates future from the past
Might is the past tense of May
Progressive tense indicates an ongoing action in the past, present or future
The perfect progressive form indicates an action that is ongoing, but will be completed at some definite time
Infinitive verbs: To + verb. This is considered as the most basic form of the verb, or the building block of all other tenses. To form
other tenses of the verb, you simply modify the infinitive form
- Avoid sentences that insert a word (or words) between to and the verb (rarely tested, but almost always incorrect)
IF & THEN CLAUSE

IF Clause THEN clause Example

Present will + base verb If she wins the lottery, she will give half the money to charity

Past would/could + base verb If she won the lottery, she would give half the money to charity

Future would/could + have + past If she had won the lottery, she would have given half the money to
participle charity
** Conditional words would and could NEVER appear in the IF clause
VERB TENSE - PRESENT TENSE
Tense When to Use How to Use
Simple present Generally expresses events, or situations that have existed in the past, exist now (in the
present) and will probably exist in the future
- I understand what you are saying

General truths
- Grass is green

Present perfect If an event started in the past but continues (or remains true in) in the present / Has/Have + Past participle.
actions have not yet finished Past participle of a regular verb is
- We have lived in a little hut for three days verb + -ed ending (example:
- Our country has enforced strict immigration laws for thirty years walked, danced, jumped)

When an event occurred at an indeterminate point in the past
- They have known each other for the longest time
- I have been to California

If the time period has not finished
- I have seen three movies this week


Giving recent news
- Michael has crashed his car again

Present perfect Recent activity


continuous - She's been working hard recently

Emphasis on the duration or length of an activity


- Jack has been painting for 4 hours

Recently finished activity with a present result


- I've been working in the garden, that's why my hands are so dirty

Present If an event is occurring at this very moment / around now / near future (especially planned Am/is/are + verb-ing
continuous activities)
- The kids are watching TV
- We aren't working hard these days
- Polly is coming for dinner tomorrow
VERB TENSE - PAST TENSE
Tense When to Use How to Use
Simple past If an event started and finished at a particular point of time in the past
- Molly and Rita embraced at the airport
- George ate all his dinner
Past perfect If more than one event in the sentence occurred at different times in the past, use the Had + past participle
past perfect tense for the earlier action (FINISHED action) and simple past for the
later action
- The film had started by the time we arrived at the theatre
- The teacher thought that Johnny had cheated on his exam

Past perfect Past perfect progressive is used to express CONTINUOUS activity up to a specific point of
continuous time in the past
- They had been waiting for 2 hours before their friends finally arrived

Past continuous Duration in the past - actions or situations that lasted for some time in the past, and
whose duration time is unknown or unimportant
- The dog was barking

Interrupted actions in progress when one action in progress is interrupted by another


action in the past. We usually use when or while to link these two sentences.
- I was talking with James when the telephone rang


Actions in progress at the same time in the past - two or more activities happening at
the same. We usually use when or while to link the two sentences
- When Bob was painting windows, Mary was working in the kitchen
VERB TENSE - FUTURE TENSE
Tense When to Use How to Use
Simple future Promises
- I promise I will buy you this toy

Unplanned actions (spontaneous decisions)


- Don't worry! I will help you with this problem

Predictions based on experience or intuition


- It will rain in a moment

Habits (obstinate insistence, usually habitual)


- She will bite her lip if she is thinking or if she's nervous about something

** Remember, you should never use will to say what somebody has already arranged or
decided to do in the future:
Correct: Mike is moving to New York next week
Incorrect: Mike will move to New York next week

Future perfect The Future Perfect expresses the idea that something will occur before another action in the Will have + past participle
future. It can also show that something will happen before a specific time in the future
- I will have been in London for six months by the time I leave Am/is/are + going to have +
- Sam is probably going to have completed the proposal by the time he leaves this past participle
afternoon

Incorrect: By the time I write to Leo, he will probably move


Correct: By the time I write to Leo, he will probably have moved
Future perfect Emphasize the length of time or duration of an event occurring before and up to another
continuous event in the future
- John will have been studying for 6 years by the time he finishes his exam

Future continuous Future actions in progress Will + be + verb-ing


- In an hour, I will be sitting in front of my TV

Guesses about the present/future
- He won't be coming any time soon. He is still at the office

Polite questions about somebody's intention


- Will you be going to the supermarket? I have something to buy
MOODS
INDICATIVE
Verbs in indicative mood deal with real events

SUBJUNCTIVE
Verbs in subjunctive mood deal with events that are not necessarily true

Hopes, dreams, desires and requests That and That Parallelism


IF CLAUSE
formed with the word that is to be maintained
It is the desire of one person or body for
another person or body to do something.
When the if clause It is unclear as to whether or not the
represents something second person or body will actually do
contrary to reality - In these what is asked
Incorrect: He said that he
cases, the verb is always
Subjunctive is formed with that + base had studied well and he
were, regardless of the
form of the verb and follows words would score well
subject
such as advice, advisable, ask, arrange, Correct: He said that he had
better, demand, desire, instruct, studied well and that he
If he were tall, he would be
instructions, intend, intentions, order, would score well
able to play basketball
better pray, prefer, propose, recommend,
suggest, request, require, urge, urgent,
vital etc. The base form of the verb is
always plural

Correct: The parolee knew it was


imperative that he find a job quickly
(base form of the verb)
Incorrect: It is imperative that she
must sign the permission slip (must is
redundant)
Correct: It is imperative that she sign
the permission slip
VOICE

ACTIVE VOICE PASSIVE VOICE


The subject of the sentence has an action
performed on it by someone or something
else
- The pizza was eaten by the hungry
students

Passive voice is formed with a form of the


verb to be, followed by a participle. The
person or people performing the action in
SUBJECT PERFORMS THE ACTION
the sentence almost always follow the verb
- The hungry students ate the pizza
Required when the non-underlined portion
of the sentence contains the person or
agent performing the action preceded by
the word by
Incorrect: The shuttle launch seen
around the world by people of all ages,
all races and all religions (missing a verb,
and therefore is a fragment)
Correct: The shuttle launch was seen
around the world by people of all ages,
all races and all religions

Active voice is preferred to passive voice. Only transitive verbs (verbs that take direct objects) can be
written in passive voice
- The aliens arrived on Neptune in the 20th century
PRONOUNS

Subject Pronouns Object Pronouns Possessive


Pronouns Who vs. whom
I Me My, mine
To simplify who/whom questions, try rearranging the
You You Your, yours sentence into a question, and then answer it
He Him His
Question: Who/m did Kate marry?
She Her Her, hers
Answer: Kate married him
It It Its (not its)
We Us Our, ours Since the pronoun used in the answer is "him," an
object pronoun, the pronoun in the original sentence
Example They
#1 Them Their, theirs should also be an object pronoun: whom
Incorrect:
Who Janice and ME went on
Whom a picnic together Whose
Correct: Janice and I went on a picnic together
The pronoun is part of the subject, and hence, the correct form is
I and not me
Example # 2
Incorrect: The picnic was attended by Janice and I
Correct: The picnic was attended by Janice and me Adding a pronoun in an answer choice more often
Picnic is the subject, while the pronoun is part of the object, thus than not makes the answer choice wrong. However,
the correct form is me and not I sometimes it is essential to introduce a pronoun to
add clarity
Example # 3
Marston was an early seventeenth century dramatist and it is likely The first step to solving a pronoun question is to
that him and Shakespeare borrowed ideas from one another. identify all the pronouns
It is likely that him and Shakespeare borrowed ideas from one
another A relative pronoun (i.e., that, which, who etc.) must
It is likely that they borrowed ideas from one another refer to the word immediately preceding it
It is likely that him and Shakespeare borrowed ideas from each
other A single pronoun can only refer to a noun, not a
It is likely that himself and Shakespeare borrowed ideas from one noun and a verb
another
It is likely that he and Shakespeare borrowed ideas from one
another
Explanation: This question tests pronoun agreement. The pronoun
him in the original sentence replaces Marston. It is in the wrong
case. Instead of the objective case, the pronoun should be in the
subjective case, since Marston is the subject of the sentence.
Therefore (E), which uses the subjective pronoun he, is correct. (B)
PRONOUNS 1

A pronoun must refer to one and ONLY one Pronoun must agree in number
antecedent (noun) (singular/plural) with the antecedent

Implication is not enough; there must be a stated After finding the antecedent, check whether the
antecedent for every pronoun pronoun agrees in number

Friendship was something that James always valued, ** The deadly four: it, its, they, their
so he disliked it when THEY talked about him behind
his back Incorrect: Police work is very important as THEY
help to enforce the laws of the state
He, him and his clearly refers to James. However, Correct: Policemen are very important as THEY
the pronoun THEY has no antecedent at all. One help to enforce the laws of the state
might think that THEY refers to friends, but the word Correct: Police work is very important as IT is the
friends is never mentioned in the sentence, only the backbone of the state
word friendship is. Therefore, the sentence is
incorrect Example # 1
Incorrect: When the person calls, take down
THEIR phone number
Correct: When the person calls, take down HIS
phone number
Correct: When the people call, take down THEIR
phone number

Example # 2
Incorrect: Everyone here will need THEIR own
pencil
Correct: Everyone here will need HIS own pencil
PRONOUNS 2

POSSESSIVE POISON IMPERSONAL PRONOUNS ONE / YOU

Possessive pronouns can refer back to Never use one or ones to refer back to any
possessive nouns. However, subject and antecedent except one. Similar for you
object pronouns may NOT refer back to
possessive nouns. Subject and object pronouns Example # 1
may only refer back to subject and object nouns Incorrect: A person should leave a light on in an
empty house if one wants to give the impression
Incorrect: Joses room is so messy that his that someone is at home
mother calls HIM a pig Correct: Incorrect: A person should leave a light
Correct: Joses room is so messy that his mother on in an empty house if he or she wants to give
calls Jose a pig the impression that someone is at home
Correct: One should leave a light on in an empty
house if one wants to give the impression that
someone is at home

Example # 2
Incorrect: One should have their teeth checked
every six months
Correct: One should have one's teeth checked
every six months
Correct: You should have your teeth checked
every six months
MODIFIERS

A modifier or a modifying phrase describes someone or


something in the sentence
A modifier must always be placed as close as possible to the word
it's modifying

ADJECTIVES

An adjective describes ONLY a noun or


pronoun and answers the questions: how ADVERBS
many, which one, what kind?
An adverb usually modifies a verb, but it can
The following sense verbs (verbs that also describe an adjective, another adverb, a
describe someone's sensation or feeling or preposition or a phrase. Many adverbs are
perception) require adjective modifiers: formed by adding ly to the adjective
- Sound
- Look Incorrect: My friend is a real interesting
- Smell person
- Taste Correct: My friend is a really interesting
- Feel person
- Seem The adverb really must be used to modify
the adjective interesting
The smart man acts quickly
The adjective smart modifies the noun man,
while the adverb quickly modifies the verb
acts

Incorrect: After she returned from the three-


week vacation, she looked very well
(looked is a sense word, well is a
adverb)
Correct: After she returned from the three-
week vacation, she looked very good
(adjective)
MODIFIERS 1

MODIFIER ERRORS

DANGLING MODIFIERS MISPLACED MODIFIERS POSSESSIVE POSION

When the modified noun is not present This occurs when the modified noun is not Incorrect: Unskilled in complex
placed directly next to the modifying phrase. math, Bills score on the entrance
Incorrect: Using the latest exam was poor
technology, the mechanical A modifying phrase must not be separated from Correct: Unskilled in complex
problem was identified quickly the noun it modifies math, Bill did not score well on
The modifier, using the latest the entrance exam
technology, is probably describing A modifying phrase that begins a sentence
the technician who identified the refers to the noun or pronoun immediately
problem. However, a technician following the phrase
never appears in the sentence
Correct: Using the latest technology, Example # 1
the engineer identified the problem Incorrect: Upon leaving the register, the
quickly cashier handed the customer a receipt
The modifier, upon leaving the register seems
to modify the cashier, although it should
modify the customer. In order to correct this,
we must place the modifying phrase directly
next to what it modifies
Correct: Upon leaving the register, the
customer received a receipt from the cashier

Example # 2
Incorrect: Kendra is happy, like all her
friends, to be on vacation
Correct: Kendra, like all her friends, is happy
to be on vacation
MODIFIERS 2

RELATIVE PRONOUNS - which, that, where, who, whose, whom


Relative pronouns introduce relative clauses, which are a type of dependent clause. Relative clauses modify a word, phrase, or
idea in the main clause

Who introduces phrases that modify a person or a group of people (if the antecedent is in the
object, then whom is to be used)

Which introduces phrases that modify things

That can be used to modify either people or things

Where can be used to modify to places

In which = places/ situations/time period/ phenomenon

Example
Incorrect: The Yankees, never liking to lose, practice everyday
Correct: The Yankees, who never like to lose, practice everyday
Correct: One of the books, which were on the table is mine

That is preferred to which


That is typically used to refer to singular nouns, and those/these is used to refer to plural nouns
,which ,that ,who ,whom ,it, whose ,it - refers to the immediately preceding noun, and not to the action of the entire
preceding clause
Example
The police found the murder weapon, which made the prosecutors job much easier
The above sentence literally means that the murder weapon itself rather than finding the murder weapon made
the prosecutors job easier
Better: The police found the murder weapon making the prosecutors job much easier
MODIFIERS 3
ESSENTIAL VS. NON-ESSENTIAL MODIFIERS

Which is used to introduce non-essential modifiers can be removed from the sentence without the
sentence losing any essential meaning

That is used to introduce essential modifiers essential to the meaning of the sentence

Commas are used to separate non-essential modifies from the noun that is modified

Non-essential: To find my house, walk down the left side of the street until you reach the third house, which is
red
Essential: To find my house, walk down the left side of the street until you reach the third house that is red

Non-essential: This is my uncle John, who lives in Toronto


Essential: This is my uncle John who lives in Toronto

Only guests who are accompanied by tenants may use the pool
Only guests, who are accompanied by tenants, may use the pool
The first sentence identifies a specific group of guests who may use the pool: those accompanied by tenants (as
opposed to those unaccompanied by tenants). By contrast, the second sentence indicates (somewhat illogically)
that only guests (as opposed to tenants) may use the pool, and that they just happen to be accompanied by
tenants. The second sentence contains a non-essential clause where an essential clause is required

AND/BUT/ALTHOUGH

When there is an and/but/although in a sentence, then the pronoun refers to the first noun or the main
subject
Although Mary is younger than Susan, she is more mature
PARALLELISM 1
PARALLEL STRUCTURES

Nouns Trevor collects stamps, coins and cards

Adjectives The wait staff was prompt, friendly and competent

Modified nouns A positive attitude can lead to both practical success and spiritual fulfillment

Verbs We worked all day, ate all evening, and slept all night

Verb infinitives I decided to swim across the river than sail around the world (the second to is optional)

Participial phrases The rain continued to fall, providing water for thirsty plants but flooding the streets as well

Adverbs Ive noticed that you often howl angrily after you cower fearfully

Adverbial phrases Ive noticed that you often howl in anger after you cower in fear

If one item includes a pronoun, Only structures that are logically CAUSE EFFECT PARALLELISM
it is often appropriate to include parallel must be structurally
the same pronoun in all items parallel Whenever there is a cause effect in
a sentence, the effect must begin
Incorrect: I prefer to hire Correct: Ken traveled around the with an ing form of the verb after
employees who work hard to world, visiting historic sites, the comma
those that dont eating native foods, and
Correct: I prefer to hire learning about new cultures The stock markets are strong,
employees who work hard to Incorrect: Ken traveled around the giving us better forecasts
those who dont world, visited historic sites, ate The rains were heavy, leading to
native foods, and learned about some crop damage
new cultures The principal declared the results,
The incorrect version gives all making everyone cheer
activities equal emphasis, instead
of making the last three activities
subordinate to the main activity of
traveling around the world
PARALLELISM 2
Verbs of Being Be aware Idioms with built-in parallel structure the word immediately after the
first part of the pair will be the same as the word immediately
Instead of expressing what the following the second part of the pair (Example: Not only in but also
subject does, these verbs express in.. )
what a subject is, or the condition a
subject is in More X than Y
The more X the greater Y
Most common form of the verb being No less was X than was Y
is to be As X to Y
Not only X but also Y
Forms of to be are is, am, are, Not X but rather Y
was, were, been, being X instead of Y
The same to X as to Y
Other common words of being Range from X to Y
appear, become, feel, grow, look, Both X and Y
remain, seem, smell, sound, stay, Mistake X for Y
taste, turn Prefer X to Y
X regarded as Y
Incorrect: The flower bouquet To think of X as Y
WAS the husbands giving of love Believe X to be Y
to his wife Between X and Y
Correct: The flower bouquet WAS Either X or Y
the husbands loving gift to his As X as Y
wife Just as X so Y (Example: Just as gills are to fish, so lungs are to humans)
The two sides of the being word Not so much X as Y (Example: Not so much to show Jane up as to make her
was are flower bouquet and appear foolish, Sarah pointed out Janes error to their supervisor)
husbands giving of love. In order More/greater/fewer less than
to achieve parallelism, rewrite the Same X as Y
Articles are parallel
sentence a,
replacing an, the
giving with Just as X, so to Y (Example: Just as stealing is frowned upon, so too is
the noun gift cheating)
COMPARISON 1
Category Example

like, unlike, likening, more than, greater than, less than, shorter than, different from, as, as (adjective)
Comparison
as, as many as, as few as, as much as, as little as, as high as, as short as
signals

Like should be used to compare people or things (any nouns)


Incorrect: Bella and June, AS their mother, are extremely smart
Correct: Bella and June, LIKE their mother, are extremely smart

Like vs. As
As/as if should be used in a comparison involving clauses (any phrase that includes a verb)

Incorrect: Just LIKE swimming is good exercise, skiing is a great way to burn calories
Correct: Just AS swimming is good exercise, skiing is a great way to burn calories


Incorrect: Franks build, like his brother, is broad (here, franks build is being compared to his
brother. This is not logical as it does not compare two similar things)
Correct: Franks build, like that of his brother, is broad

Incorrect: The host paid more attention to his business clients than others
Parallelism must
Correct: The host paid more attention to his business clients than to others
compare similar

things and be
Incorrect: Mary is taller than any girl in class (this implies that Mary is even taller than
structurally
herself)
parallel
Correct: Mary is taller than any other girl in class

Incorrect: I enjoy flying planes more than I like to drive a car
Correct: I enjoy flying planes more than I like driving cars
COMPARISON 2
Category Example
Incorrect: Byron admired Dryden more than Wordsworth
Unclear
Correct: Byron admired Dryden more than he did Wordsworth
comparisons
Correct: Byron admired Dryden more than Wordsworth did
Incomplete comparisons are normally corrected by inserting a phrase like those of, those in, those at,
that of, that in, and that at
Incomplete
comparisons Incorrect: The peaches here are riper than other fruit stand (peaches are being compared with an
unlike thing fruit stands)
Correct: The peaches here are riper than those at other fruit stand

Compare similar things. The word after like/unlike must match the word after the comma

Like/Unlike
Like Mary, John has a black car
Unlike Mary, Susan is tall
Compared to (comparison between unlike things) vs. compared with (comparison between like things)

Compared to vs.
He compared her to a summer day
compared with
The police compared the forged signature with the original

Use comparative form when comparing two things; the superlative form when comparing 3 or more things
IDIOMS 1
Correct Usage Incorrect Usage
Awareness of Awareness about
To regard X as Y To regard X as to regard Y
So X as to Y (emphasis is on X)

Example: So tall as to touch the ceiling
Enough X to Y (emphasis is on Y)

Example: Tall enough to touch the ceiling
Considered/consider Considered to be
Example: Scientists consider control factors an integral Example: Scientists consider control factors to be an
element integral element
Declare XY Declare X to be Y
Example: Declared the Roman aqueduct of Segovia a Heritage Example: Declared the Roman aqueduct of Segovia to be a
of Humanity in 1985 Heritage of Humanity in 1985
In the vicinity of Around the vicinity of
Prices can only be higher or lower This price is more expensive than that one
Agree with another person

Agree to/upon something
In contrast to In contrast with
Regarded as Regarded to be
When rates refer to the price charged, it should be followed
by for

Example: Rates for telephone service have increased in recent
years.
Thinking words such as theory, belief or believe are often
followed by that
Example: Lucys belief that the Holocaust did not occur is
misguided
Persuade X to do Y Persuade X from doing Y
IDIOMS 2
Correct Usage Incorrect Usage

Apprenticeship as Apprenticeship of being

Dispute over Dispute about / dispute concerning

Attempted to Attempted

transparent enough to be
transparent enough so as to be
transparent as to be

Estimated to be Estimated at

Forbid X to do Y Forbid X from doing Y

Prefer X to Y Prefer X over Y

Consider XY
Consider X as Y
Many financial analysts consider an upward trend in a firms current
Many financial analysts consider an upward trend in a firms current
ratio a sign of improving liquidity
ratio as a sign of improving liquidity

Remembered for Remembered because of


ODDS AND ENDS 1
RULE # 1 Countable vs. uncountable
Countable items include dollars, hats, people and buildings
Uncountable items include money, water, wreckage and patience
- Always takes a singular verb in a sentence

Countable modifiers Uncountable modifiers Examples

Many Much

As many as As much as
Incorrect: The company fired no less than fifty
employees
Few/fewer Little/less
Correct: The company fired no fewer than fifty
employees
Number of Amount of

RULE # 2 Among vs. Between

Relating to two things Relating to three or more things

Between X and Y Among X, Y and Z

X is better than Y X is the best (among x, y, and z)

X has more than Y X has the most (among x, y and z)

X has less than Y X has the least (among x, y and z)


ODDS AND ENDS 2

Increase and Increase and decrease represent the change of ONE The price of silver increased by 10 dollars
decrease vs. thing over time The price of silver is greater than the price of
greater and less copper
Greater or less signals a comparison between TWO
things

Greater than is correct only when it is used to


describe numbers alone (greater than 10). If a sentence
measures a percent rather than solely a number, the
correct option is more than

Connecting words Be aware of sentences that have no logical connectors Incorrect: I need to relax, I have so much to do
between 2 independent clauses. Common connecting (run-on sentence)
words are: and, or, nor, but, yet, although, when, because, Correct: I need to relax, but I have so much to do
for, since, before, after, if, & unless
Connecting The semicolon ( ; ) is used to connect two closely Incorrect: Andrew and Lisa are inseparable;
punctuation related statements. BOTH statements need to stand doing everything together (second part of the
together as independent sentences sentence is incapable of standing on its own)
Correct: Andrew and Lisa are inseparable; they
The colon ( : ) is used to equate two parts of a do everything together
sentence, where the second part is dependent on Incorrect: I love listening to: classical rock, rock
the first part. It is often used to equate a list with its and pop music
components. One should be able to insert the word Correct: I love many kinds of music: [namely]
namely after the colon. Only the statement that precedes classical rock, rock and pop music
the colon must be able to stand alone

If/Whether Whether is correct when you're discussing two options
(whether to get chocolate or strawberry ice cream). On
the GMAT, whether will (almost) always beat if

Only/once Words with only/once always refer to the item coming Incorrect: Existed once
immediately after them Correct: Once existed
ODDS AND ENDS 3

Because vs. In that When because and in that are in a sentence, more Teratomas are unusual forms of cancer because they are
often than not, in that would be correct. In that composed of tissues such as tooth and bone not normally
qualifies the previous sentence, while because is just found in the organ in which the tumor appears.
used to show a simple causal relationship
A. because they are composed of tissues such as tooth
and bone
B. because they are composed of tissues like tooth and
bone that are
C. because they are composed of tissues, like tooth and
bone, tissues
D. in that their composition , tissues such as tooth and
bone, is
E. in that they are composed of tissues such as
tooth and bone, tissue
Can vs. Could If you are just assuming something, could should be Artificial intelligence emerged during the late 1950's as
used an academic discipline based on the assumption that

computers are able to be programmed to think like
Could is used for: people
- Possibility (John could be the one who stole the
money) A. are able to be programmed to think like people
B. were able to be programmed to think as people
- Condition (If I had more time, I could travel around
the world) C. can be programmed to think as people can
D. could be programmed to think like people
- Suggestion (You could spend your vacation here)
E. are capable of being programmed to think like
- Polite request (Could I have something to drink?) people do

Usual vs. Is usual When something is compared to a subgroup to which it Incorrect: A Mercedes is more expensive than usual
belongs, is usual should be used. When something is for a car
compared to itself, usual is fine Correct: A Mercedes is more expensive than is usual
for a car
He is nicer than usual
The If we use the we are saying that there are only 50
Although about 99% of the more than 50 million Turks
million Turks in the whole world; if we dont use the
THINGS TO AVOID
Noun forms (words ending in -ance, -ence, -ment, -ion,-ity)
Because of
Apostrophe
Passive voice

WORDS/PHRASES THAT ARE ALMOST ALWAYS INCORRECT


Do it (use do so)
- Incorrect: He said that I ate his cookies, but I didnt do it

The numbers of
- Incorrect: The politicians were amazed at the numbers of anti-war protestors

Whether or not (just use whether)


- Incorrect: He couldnt decide whether or not to apply to Harvard

Due to (should only be used if it can be replaced by caused by; used to modify a noun)
- Incorrect: the game was postponed due to rain
- Correct: His failure was due to his laziness
- Incorrect: He failed due to his laziness

So as to (use such that)


Just like
On account of
Occurring (to be used only for natural calamity)
Such like
Owing to
Lesser
Hopefully
Seeming
Not any
Being/ having
CRITICAL REASONING
IMPORTANT NOTES
When reading Logical Reasoning stimuli, you should seek to make several key determinations:
Does the stimulus contain an argument or is it only a set of factual statements?
If the stimulus contains an argument:
What is the conclusion?
Is the argument strong or weak? To determine the strength of the argument, consider the relationship
between the premises and the conclusiondo the premises strongly suggest that the conclusion
would be true? Does the conclusion feel like an inevitable result of the premises? Or does the
conclusion seem to go beyond the scope of the information in the premises? DO THE FACTS
SUPPORT THE CONCLUSION?
Read the fine print!
Quantity and Probability Indicators
Scope
Carefully read and identify the question stem. Do not assume that certain words are automatically
associated with certain question types
One of the most effective techniques for quickly finding correct answer choices and avoiding incorrect
answer choices is pre-phrasing. Pre-phrasing an answer involves quickly speculating on what you expect
the correct answer will be based on the information in the stimulus
INFER = MUST BE TRUE
STIMULUS

PREMISE ARGUMENT
A fact, proposition, or statement from Premise + Conclusion
which a conclusion is made Conclusion is a statement or judgment
that follows from one or more reasons
Premises support and explain the Conclusion = premise + assumption
conclusion (implicit)
Definitio Example: All professors are ethical. Mason
n Example: The Jacksonville area has just is a professor. So Mason is ethical.
over one million residents. The Cincinnati
area has almost two million residents. The The first two statements in this argument
New York area has almost twenty million give the reasons (or premises) for
residents accepting the third statement, which is the
conclusion

because, since, for, for example, for the


reason that, in that, given by, as
indicated by, due to, owing to, this can be
seen from, we know this by
Indicator thus, therefore, hence, consequently, as a
s Additional premise indicators result, so, accordingly, clearly, must be
(another premise that supports the that, shows that, conclude that, follows
conclusion but is sometimes non- that, for this reason
essential to the conclusion)
furthermore, moreover, besides, in
addition, whats more

Imp: Premises and conclusions can be presented in any order; the order has no
effect on the logical structure of the argument. Similarly, it is not necessary for
the premise and conclusion to be split into multiple sentences
Tip: The conclusion is generally the first or last sentence of a stimulus
DETERMINING PREMISES AND CONCLUSIONS
Example Premise Conclusion (may be added in the
question stem )
The rapid diminishment of the ecosystem of Premise 1: The rapid diminishment of the We must take immediate steps to
the Amazon threatens the entire planet. ecosystem of the Amazon threatens the entire convince the Brazilian government that
Consequently, we must take immediate planet. planned development projects need to be
steps to convince the Brazilian government curtailed
that planned development projects need to Premise 2: for the simple reason that these
be curtailed for the simple reason that these development projects will greatly accelerate
development projects will greatly accelerate the loss of currently protected land.
the loss of currently protected land.
Every professor at Fillmore University Every professor at Fillmore University teaches Fillmores Professor Jackson, therefore, is
teaches exactly one class per semester. exactly one class per semester teaching exactly one class this semester
Fillmores Professor Jackson, therefore, is
teaching exactly one class this semester. The first sentence is the main proof offered by
Moreover, I heard Professor Jackson say she the author for the conclusion. The third
was teaching only a single class sentence begins with the additional premise
indicator moreover. The premise in this
sentence is non-essential to the argument, but
provides additional proof for the conclusion and
could be, if needed, used to help prove the
conclusion separately (this would occur if an
objection was raised to the first premise)
Humans cannot live on Venus because the The surface temperature is too high Humans cannot live on Venus
surface temperature is too high
Therefore, since higher debt has forced Higher debt has forced consumers to lower Banks now have less money to loan
consumers to lower their savings, banks now their savings
have less money to loan
Television has a harmful effect on society. Premise 1: This can be seen from the poor Television has a harmful effect on society.
This can be seen from the poor school school performance of children who watch
performance of children who watch significant amounts of television
significant amounts of television and from
the fact that children who watch more than Premise 2: and from fact that children who
six hours of television a day tend to read watch more than six hours of television a day
less than non-television watching children tend to read less than non-television watching
children
TYPES OF CRITICAL REASONING QUESTIONS

PROVE HELP HURT


Must accept the stimulus even if Reasoning errors are often present in Reasoning errors are often present in
it contains an error in reasoning and the stimulus; you may use the new the stimulus; you may use the new
use it to prove that one of the answer information provided in the answer information provided in the answer
choices must be true choices to strengthen/support the choices to weaken the argument
argument
Any information in an answer choice
that does not appear either directly in
the stimulus or as a combination of the
items in the stimulus will be incorrect

Must be true/ Inference Assumption Weaken

Main point Strengthen/support

Method of reasoning Paradox

Flaw in reasoning

Parallel reasoning
A. MUST BE TRUE/ INFERENCE
Paraphrased answers - answers that Things to keep in mind
restate a portion of the stimulus in different - The stimulus comprises a set of facts ONLY,
terms. Because the language is not exactly and the objective is to determine the
the same as in the stimulus, paraphrased conclusion based on the facts presented. NO
CORRECT answers can be easy to miss ADDITIONAL ASSUMPTIONS MUST BE MADE
ANSWERS - Be careful of indicator and modifier words such
Combination answers - Answers that result as entirely, some, almost, all, could lead to,
from combining two or more statements in many , not, but . They define the scope (narrow
the stimulus or broad) and the tone of the argument.

INCORRECT ANSWERS
Answers that Because the criteria in the question stem requires you to find an answer choice that Must Be True, answers that
are possible only could be true or are even likely to be true are incorrect. These answers are attractive because there is nothing
but not demonstrably wrong with them (for example, they do not contain statements that are counter to the stimulus)
certain
Exaggerated These answers take information from the stimulus and then stretch that information to make a broader statement
answers that is not supported by the stimulus

For example, if the stimulus stated that Some software vendors recently implemented more rigorous licensing
procedures, an incorrect answer would exaggerate one or more of the elements: Most software vendors recently
implemented more rigorous licensing procedures.
Answers that For example, if the stimulus discusses the economic policies of Japan, be careful with an answer that mentions U.S.
present so- economic policy. Look closely at the stimulusdoes the information about Japanese economic policy apply to the
called new U.S.?
information
Do not eliminate if the answer falls within the sphere of a term or concept in the stimulus, or if it is a consequence
of combining stimulus elements
Reverse The stimulus might state, Many people have some type of security system in their home.
answers An incorrect answer then reverses the elements: Some people have many types of security systems in their
home.
Shell Game Occurs when an idea or concept is raised in the stimulus and then a very similar idea appears in the answer choice,
but the idea is changed just enough to be incorrect but still attractive
Opposite Opposite of the stated facts of the stimulus. Example, stimulus shows that X caused Y, but answer choice states
answer that Y caused X
When a stimulus contains only the opinions of others, then in a Must Be True question you can eliminate any answer choice that
MUST BE TRUE CONTINUED
Flavonoids are a common component of almost all plants, but a specific
variety of flavonoid in apples has been found to be an antioxidant.
Antioxidants are known to be a factor in the prevention of heart disease

Which one of the following can be properly inferred from the passage?
A. A diet composed largely of fruits and vegetables will help to prevent
heart disease
B. Flavonoids are essential to preventing heart disease
C. Eating at least one apple each day will prevent heart disease
D. At least one type of flavonoid helps to prevent heart disease
E. A diet deficient in antioxidants is a common cause of heart disease

1st Statement: Flavonoids are a common component of almost all plants


2nd Statement: A specific variety of flavonoid in apples has been found to
be an antioxidant
3rd Statement: Antioxidants are known to be a factor in the prevention of
heart disease
A The answer choice could be true, but it is too broad to be supported by
the facts: nowhere are we told that a diet of fruits and vegetables will
help prevent heart disease. Perhaps apples are the only fruit with the
antioxidant flavonoid and there is nothing beneficial about other fruits
and vegetables. And, eating a diet of fruits and vegetables is no
guarantee that the diet includes apples. Regardless, this answer choice
can be especially attractive because it plays on the general perception
that fruits and vegetables are good for you
B Nothing in the stimulus supports the rather strong statement that
flavonoids are essential to preventing heart disease
C This answer is incorrect because the language is too strong: the stimulus
only stated that apples contain an element that was a factor in
preventing heart disease, not that they definitely will prevent heart
disease
D This answer is the closest to our pre-phrase, and this is the correct
answer. Notice how the language of this answer choicehelps to
preventmatches the stimulus languagefactor in the prevention.
E This answer choice also could be true, but it cannot be correct because
the stimulus makes no mention of the causes of heart disease. Just
because an antioxidant can help prevent heart disease does not mean
B. MAIN POINT

Which one of the following most accurately expresses the main conclusion of the argument?
Question
identifiers Which one of the following most accurately restates the main point of the argument

Primary
To identify the CONCLUSION based ONLY on facts/premises explicitly listed in the stimulus
objective

Answers that are true but do not encapsulate the authors point
Incorrect
answers
Answers that repeat premises of the argument

Example:
Journalist: A free marketplace of ideas ensures that all ideas get a fair hearing. Even ideas tainted with prejudice and malice can prompt beneficial outcomes. In
most countries, however, the government is responsible for over half the information released to the public through all media. For this reason, the power of
governments over information needs to be curtailed. Everyone grants that governments should not suppress free expression, yet governments continue
to construct near monopolies on the publication and dissemination of enormous amounts of information

Which one of the following most accurately expresses the conclusion of the journalists argument?
(A) The freedom of the marketplace of ideas is in jeopardy.
(B) Preserving a free marketplace of ideas is important
(C) The control that governments have over information needs to be reduced
(D) Ideas that have malicious content or stem from questionable sources can be valuable
(E) Governments have near monopolies on the dissemination of many kinds of information

(F) The author would agree with this statement but this is not the Main Point of the argument; rather, it is closer to a premise that might support the conclusion
(G) The author believes that the freedom of the marketplace of ideas is at risk, and in stating that we should curtail the governments power over information,
the author assumes that preserving a free marketplace of ideas is important. Thus this answer choice would be better described as an unstated premise
that supports the conclusion
(H) This is the correct answer. Remember, any answer that is a paraphrase of the conclusion of the argument will be the correct answer to a Main Point question
(I) The stimulus specifically notes that malicious or prejudicial ideas can prompt beneficial outcomes. The outcome of an idea is different than stating the
ideas themselves can be valuable.
(J) The stimulus states that governments continue to construct near monopolies on the publication and dissemination of enormous amounts of information.
This phrasing is not the same as answer choice (E), which asserts that the government already has a monopoly on the dissemination of many kinds of
information
C. WEAKEN
Question Which of the following most seriously undermines the argument?
Identifiers Which of the following, if true, calls into question the validity of the argument?
Trigger words: Weaken, attack, undermines, contradicts, evidence against, challenge, damage, counter, refute
etc.
Primary Objective There is always a flaw in the reasoning of the stimulus (gross generalization, conclusions drawn from incomplete
information, improper comparison etc.)

To solve these questions, you first need to identify the premise and the conclusion. In this question type, we
assume an answer choice presented to be true even if it introduces new information (obviously, the information
has to be relevant to the stimulus)
Incorrect answers Opposite answers answers that strengthen rather than weaken the argument
Shell game answers similar idea to that of the stimulus, but not entirely true
Improper comparison comparing two or more items that are essentially different
Out of scope answers
Wrong tone answers
Reversal of causality/incorrect causality
Cause & Effect - Causality occurs when one event is said to make another occur. Just because one event occurs before the
Reasoning other, or that two events occur simultaneously, does not mean that there is a cause-effect relationship
- Words that indicate cause-effect relationship: cause by, because of, responsible for, reason for, leads to,
induced by, promoted by, determined by, produced by, product of, played a role in, was a factor in, is an effect
of
- The question assumes that the only cause is the one stated in the conclusion and that there are no other
causes that can create that particular effect
- In weaken questions, the correct answer choice with do any of the following:
Find an alternate cause for the stated effect. Because the author believes there is only one cause,
identifying another cause weakens the conclusion
Show that even when the cause occurs, the effect does not occur
Show that although the effect occurs, the cause did not occur
Show that the stated relationship is reversed (the claimed effect is actually the cause of the claimed
cause)
Show that a statistical problem exists with the data used to make the casual statement

Example

The consumption of ice cream has been found to correlate with the murder rate. Therefore, consuming ice cream
must cause one to be more likely to commit murder

However, this argument fails to take into account that a third element, hot weather, could be the cause of the
increase in murder rate and in the consumption of ice cream
WEAKEN CONTINUED
Robot satellites relay important communications and identify Violent crime in this town is a becoming a serious problem.
weather patterns. Because the satellites can be repaired only Compared to last year, local law enforcement agencies have
in orbit, astronauts are needed to repair them. Without responded to 17 per cent more calls involving violent
repairs, the satellites would eventually malfunction. crimes, showing that the average citizen of this town is
Therefore, space flights carrying astronauts must continue. more likely than ever to become a victim of a violent crime.

Which of the following, if true, would most seriously weaken Which one of the following, if true, most seriously weakens
the argument above? the argument?
A. Satellites falling from orbit because of malfunctions burn A. The towns overall crime rate appears to have risen
up in the atmosphere slightly this year compared to the same period last year
B. Although satellites are indispensable in the identification B. In general, persons under the age of 65 are less likely to
of weather patterns, weather forecasters also make some be victims of violent crimes than persons over the age of
use of computer projections to identify weather patterns 65
C. The government, responding to public pressure, has C. As a result of the towns community outreach programs,
decided to cut the budget for space flights and put more more people than ever are willing to report violent
money into social welfare programs crimes to the proper authorities
D. Repair of satellites requires heavy equipment, which adds D. In response to worries about violent crime, the town has
to the amount of fuel needed to lift a spaceship carrying recently opened a community centre providing
astronauts into orbit supervised activities for teenagers
E. Technical obsolescence of robot satellites makes E. Community officials have shown that a relatively small
repairing them more costly and less practical than number of repeat offenders commit the majority of
sending new, improved satellites into orbit violent crimes in the town
A Irrelevant
A This is an opposite answer that strengthens the argument
B Doesnt provide a reason for not sending astronauts to space.
Out of scope B Because the argument is about the average citizen of this
town, information about victims of a certain age is
C Though this seems like a right answer, think about it. This is irrelevant
lots of irrelevant information, and doesnt answer our
question directly C This is the correct answer. By showing that people are more
willing to report crimes (and thus call them in for response),
D Fuel? This is a shell game fallacy. Clearly shows a statement an alternate cause for the rise in the number of calls is given
that could be true and is very attractive to the test taker, but
irrelevant to whats asked D This answer only addresses an effect of the concern over
crime, and does not address the causal relationship that
E This makes sense. If repair cost > new satellite cost, why underlies the argument
send astronauts to space to repair them? Just send new
satellites E This answer does not address a possible rise in crime or the
reasons for the rise in responses to calls involving violent
crime
D. STRENGTHEN

Question Stem would indicate some kind of support relationship strengthen, justify, help, support etc.
Identifiers
** The stimulus supports which of the following would be an inference/main point question as opposed
to the strengthen question which would point to the answer choice supporting the stimulus subtle
but important difference

Primary Identify the conclusionthis is what you are trying to strengthen!


Objective
Find the missing link between a premise and the conclusion (typically an assumption)
Important to Dont try to disprove a conclusion
keep in mind
Incorrect Opposite answers answers that weaken rather than strengthen the argument
answers Out of Scope
Shell Game
Wrong tone answers
Answers that merely state the premise

Correct answers
Connect evidence with conclusion better
Make the conclusion stronger
Arguments that contain analogies or use surveys rely upon the validity of those analogies and surveys.
Answer choices that strengthen the analogy or survey, or establish their soundness, are usually correct
Strengthen the evidence with new information (perhaps an assumption is needed to make the argument
work
STRENGTHEN CONTINUED

Dr. Larson: Sleep deprivation is the cause of many social ills,


ranging from irritability to potentially dangerous instances of
impaired decision making. Most people suffer from sleep
deprivation to some degree. Therefore we should restructure
the work day to allow people flexibility in scheduling their
work hours

Which of the following, if true, would most strengthen the


medical doctors argument about sleep deprivation?
The conclusion of the argument is straightforward. Allow
flexibility in scheduling work hours. The basic underlying
assumption in the conclusion basically tells you that the
author is assuming that greater flexibility would promote
better sleeping times. Now we can have this in our mind as
we look for answers.
A. The primary cause of sleep deprivation is overwork
B. Employees would get more sleep if they had greater
latitude in scheduling their work hours
C. Individuals vary widely in the amount of sleep they
require
D. More people would suffer from sleep deprivation today
than did in the past if the average number of hours
worked per week had not decreased
E. The extent of ones sleep deprivation is proportional to
the length of the workday
A Perfect shell game choice. So this choice is telling you that
overwork causes sleep deprivation. We are not talking about
the quantity of work here, but instead the flexibility of
scheduling. Irrelevant
B Correct
C Completely irrelevant. We are not talking about how much
sleep an individual requires at all. It presents new information
but without any connection
D This talks about a decrease in work hours and in a very
convoluted structure. Are we talking about the number of
hours worked? No. We are talking only about the flexibility of
scheduling
E. ASSUMPTIONS

Question Identifiers The conclusion depends on which of the following?


The author assumes that?
Based on
Assumption Made
Cannot be true unless?
Primary Objective An assumption is an unstated premise and is NECESSARY to arrive at a conclusion

Find the missing link between a premise and the conclusion (typically an assumption)
Incorrect answers Contain extra information - For example, lets say that an argument requires the assumption that all dogs
are intelligent. The correct answer could be that statement or even a subset statement such as all black
dogs are intelligent or all large dogs are intelligent. However, in the case of the statement all dogs and
cats are intelligent, the additional information about cats is not part of the authors assumption, and would
make the answer choice incorrect
Answers that claim an idea was the most important consideration for the author
Answer choice that repeats the premise
Difference between Strengthen questions ask you to support the argument in any way possible. This type of answer has great
Strengthen & range, as the additional support provided by the answer choice could be relatively minor or major. Speaking
Assumption in numerical terms, any answer choice that strengthens the argument, whether by 1% or by 100%, is
Questions correct

Assumption questions ask you to identify a statement that the argument assumes or supposes. An
assumption is simply an unstated premise what must be true in order for the argument to be true. An
assumption can therefore be defined as what is necessary for the argument to be true

Example: An argument concludes that a teenager is an outstanding golfer. In an Assumption question, the
correct answer could be: The teenager always hits the ball or The teenager never swings and misses the
ball. Either statement is an assumption of the argument; otherwise how could the teenager be an
outstanding golfer? In a Strengthen question, the correct answer could be: The teenager won a local club
tournament. This answer choice supports the idea that the teenager is an outstanding golfer, but does not
undeniably prove the teenager to be outstanding (what if the tournament was composed primarily of pre-
teen players?) nor is the answer an assumption of the conclusion
CORRECT ASSUMPTION ANSWER
CHOICE

SUPPORTER DEFENDER

Eliminate ideas or assertions that would undermine the


conclusion. In this sense, they defend the argument by
Links together new/ rogue elements in the stimulus
showing that a possible source of attack has been
Conclusion often contains a piece of information not
eliminated
previously included in the stimulus
Fills logical gaps in the argument
Example: People who read a lot are more intelligent than
other people. Thus, reading must cause a person to be
Example: All male citizens of Athens had the right to vote.
intelligent
Therefore, Socrates had the right to vote in Athens
Assumption: All other alternative explanations (such as
Assumption: Socrates was male
regular exercise, high-protein diet, genetics etc.) are
assumed not to exist

Watch for answers starting with the phrase at least one or at least some. For some reason, when an Assumption answer
choice starts with either of the above constructions the chances are unusually high that the answer will be correct

ASSUMPTION NEGATION TECHNIQUE


Basically involves converting the assumption question into a weaken question. The technique can only be applied to assumption
questions, so you need to be careful, but once youve eliminated other possible answers, follow this technique to check the
remaining options:
Negate the answer choice This basically asks you to assume the opposite of whatever is given in the answer choice ( I
went to the beach yesterday will become I did not go to the beach everyday last week; sweet will become not sweet; all the
missions succeeded will become not all of the missions succeeded)
** All = 100; Not All = 0-99; Some = 1-100; None = 0
Does the negated answer choice make the conclusion collapse? If the answer is yes, then the answer choice is the
right one. Any negated answer choice that attacks the conclusion or questions its validity is one that is the right answer.
F. RESOLVE THE PARADOX

Question Identifiers Which of the following, if true, most helps resolve the apparent paradox?
Resolve/ reconcile/ Explain the following Discrepancy/ contradiction/ conflict/ puzzle
Primary Objective Two contradictory facts are presented as they are. There is no inference/conclusion. You must resolve the
paradox (i.e., allow both sides to be factually correct)
Incorrect answers Answer explains only one side of the paradox
Does not conform to the specifics of the stimulus
If the stimulus contains a paradox where two items are similar, then an answer choice that explains a
difference between the two cannot be correct and vice versa. The answer choice must resolve the paradox,
not strengthen or weaken it
Correct Answers Correct answers will actively resolve the paradox, that is, it will allow both sides to be factually correct and it
will either explain how the situation came into being or add a piece of information that shows how the two
ideas or occurrences can co-exist

Park Ranger: When snowfall levels are below average during winter months, scattered patches on the forest floor often remain
exposed and accessible to scavenging wildlife. Because squirrels are able to collect nuts only in the snow-free areas of the forest,
the squirrel population tends to increase when there is a below average snowfall. However after last years unprecedented snow-
free winter season, the squirrel population in this region was determined to be a 20 year low

Which of the following, if true, helps explain the paradox above?


A. When snowfall is above average, squirrel populations tend to diminish as squirrels are unable to forage for food in snow-
covered areas
B. The squirrels spring breeding season does not begin until all of the snow in the forest has melted.
C. The red-tailed hawk, the squirrels most common predator, does not migrate south out of the forest until the first snowfall of
the winter season.
D. Forest squirrels rarely feed on berries or fruits and prefer nuts for their high calorific content
E. The current system of estimating squirrel population size is thought to be extremely accurate in its projections

A Irrelevant. The first phrase itself tells us this is useless since we are looking to explain an event that happens during a snow-free
winter
B This doesnt really explain anything either. If at all anything, the population should have gone up.
C This seems good. If the predator doesnt move until the snowfall, and there was no snowfall, then the squirrel population would
have gone down
D Irrelevant
E Irrelevant
G. BOLD FACED QUESTIONS
Question The two bold faced sentences play which of the following roles?
Identifiers
Primary Objective These questions are heavily dependent on interpreting the premise and identifying the conclusion of the
stimulus correctly and hence can be considered similar to an inference/must be true question type. To correctly
answer the question its important to identify the pattern of reasoning in the stimulus correctly. Thus, going
back to the inference lesson, for this question type, we need to isolate the conclusion, understand the premise
and identify any logical errors in the drawing of the conclusion
Incorrect answers Answer explains only one side of the bold faced questions
Does not conform to the specifics of the stimulus

Historian: Newton developed mathematical concepts and


techniques that are fundamental to modern calculus. A The first provides evidence in support of the overall position
Leibniz developed closely analogous concepts and that the historian defends; the second is evidence that has
techniques. It has traditionally been thought that these been used to support an opposing position.
discoveries were independent. Researchers have The historian doesnt defend the position, he opposes it. So
however, recently discovered notes of Leibniz that incorrect
discuss one of Newtons books on mathematics. Several
B The first provides evidence in support of the overall position
scholars have argued that since the book includes a that the historian defends; the second is that position
presentation of Newtons calculus concepts and Once again, the historian doesnt defend this position, hence
techniques and since the notes were written before incorrect.
Leibniz own development of calculus concepts and
techniques, it is virtually certain that the traditional view C The first provides evidence in support of an intermediate
is false. A more cautious conclusion than this is called for, conclusion that is drawn to provide support for the overall
however. Leibniz notes are limited to early sections position that the historian defends; the second provides
of Newtons book; sections that precede the ones evidence against that intermediate conclusion
in which Newtons calculus concepts and This is the kind of complicated wording that GMAT likes to
techniques are presented. trick us with. There is no intermediate conclusion, and the
In the historians reasoning, the two boldfaced portions historian doesnt support the overall conclusion
play which of the following roles?
D The first is evidence that has been used to support a
This is a question where its absolutely necessary conclusion that the historian criticizes; the second is evidence
to identify what the authors stand on the offered in support of the historians own position
This is true. The historian opposes the first and gives an
argument is, and hence identifying the conclusion
alternative explanation.
becomes vital. The traditional view is that Leibniz and
Newton developed these simultaneously. But people E The first is evidence that has been used to support a
believe this is false because they found Leibniz conclusion that the historian criticizes; the second is further
mentioning that hed studied Newtons book. Look at information that substantiates that evidence
what the historian says in response: A more cautious The second part of this is wrong. The first might be evidence
conclusion. So this means he doesnt believe that Leibniz
H. METHOD OF REASONING
Question The argument proceeds by
Identifiers The argument derives its conclusion by
Which one of the following is an argumentative strategy employed in the argument?
The argument employs which one of the following reasoning techniques?
Primary Objective Focus on the form of the argument instead of the concrete facts
Incorrect answers Describe an event that did not occur in the stimulus/ New Element Answers
Half right/half wrong start by describing something that occurred in the argument and end by describing
something that didnt
Exaggerated/opposite/reversed answers
A. This answer forces you to make an assessment of the premises (the
Garbage in this neighbourhood probably will not be collected
evidence) as they relate to the conclusion. Are the premises
until Thursday this week. Garbage is usually collected here
irrelevant to the conclusion? Clearly not. Therefore, this answer is
on Wednesdays, and the garbage collectors in this city are
incorrect
extremely reliable. However, Monday was a public holiday,
B. This is a half-right, half-wrong answer. The argument does establish
and after a public holiday that falls on a Monday, garbage
that one thing is likely to occur. But, is this established by ruling
throughout the city is supposed to be collected one day later
out all of the alternative possibilities? No, to do that would mean
than usual. The argument proceeds by:
presenting arguments against the garbage being collected on
(A) treating several pieces of irrelevant evidence as though
Friday, Saturday, Sunday, etc. Since this section of the answer
they provide support for the conclusion
choice does not occur, this answer is incorrect. Also, because the
(B) indirectly establishing that one thing is likely to occur by
argument does not rule out all the alternatives, the conclusion is
directly ruling out all of the alternative possibilities
not established indirectly.
(C) providing information that allows application of a general
C. This is the correct answer. Consider each piece of the argument:
rule to a specific case
providing informationa variety of information about the garbage
(D) generalizing about all actions of a certain kind on the
situation is provided. application of a general rulethe general
basis of a description of one such action
rule is that After a public holiday that falls on a Monday, garbage
(E) treating something that is probable as though it were
throughout the city is supposed to be collected one day later than
inevitable
usual. to a specific casethe specific case is the pickup of
garbage this week in this neighbourhood. Given that all elements
As usual, we begin by analyzing the structure of the
occurred and the answer presents an accurate description of the
problem:
way the author made his or her argument, this answer is correct.
Premise: Garbage is usually collected here on Wednesdays,
Now, take a moment and compare this answer to the pre- phrase
and the garbage collectors in this city are extremely reliable
you made after reading the stimulus. How similar are the two?
Premise: Monday was a public holiday
Given that you may not be familiar with the language used by the
Premise: After a public holiday that falls on a Monday,
test makers, the two may not be very similar. For example, note the
garbage throughout the city is supposed to be collected one
use in this answer of general rule to describe the last sentence of
day later than usual
the stimulus. The test makers could also have used a phrase like
Conclusion: Garbage in this neighbourhood probably will not
basic principle to achieve the same result. Your job is to match
be collected until Thursday this week
their language to what occurred in the stimulus
The argument is sound and the conclusion seems
D. This answer is an overgeneralizationa situation where one
I. FLAW IN REASONING
Question The reasoning in the argument is most vulnerable to criticism on the ground that the
Identifiers argument
The reasoning above is flawed because it fails to recognize that
The reasoning in the argument is fallacious because the argument
Primary Flaw in the Reasoning questions are exactly the same as Method of Reasoning questions with
Objective the important exception that the question stem indicates that the reasoning in the stimulus is
flawed. Because the question stem reveals that a flaw is present, you need not make a
determination of the validity of the stimulus

COMMON ERRORS OF REASONING


Causal If the causal statement is the conclusion, then the
conclusion vs. reasoning is flawed
causal claim Premise: In North America, people drink a lot of milk
made in the Premise: There is a high frequency of cancer in North
premise America
Conclusion: Therefore, drinking milk causes cancer

In this case, the author takes two events that occur


together and concludes that one causes the other

If the causal statement is the premise, then the


argument may be flawed, but not because of the
causal statement
Premise: Drinking milk causes cancer
Premise: The residents of North America drink a lot of
milk
Conclusion: Therefore, in North America there is a
high frequency of cancer among the residents

**Uncertain Use Some people claim that the values that this country - depending on the ambiguous use of a key term
of a Term or was built on are now being ignored by modern-day - it confuses two different meanings of the word solve
Concept corporations. But this is incorrect. Corporations are - relies on interpreting a key term in two different
purely profit driven enterprises, beholden only to ways
their shareholders, and as such they can only assess - equivocates with respect to a central concept
objects based on their value. - allows a key term to shift in meaning from one use to
The term value is used in the example above in two the next
COMMON ERRORS OF REASONING 2
Source Argument Attacks the person (or source) instead of the makes an attack on the character of opponents
argument they advance. Because the GMAT is
concerned solely with argument forms, a speaker can it is directed against the proponent of a claim rather
never validly attack the character or motives of a than against the
person; instead, a speaker must always attack the claim itself
argument advanced by a person.
he directs his criticism against the person making the
Here is an example: The anti-smoking views argument rather
expressed by Senator Smith should be ignored. After than directing it against the argument itself
all, Smith himself is a smoker!
A source argument can take different forms, it draws conclusions about the merit of a position and
including the following: about the content of that position from evidence about
1. Focusing on the motives of the source the positions source
2. Focusing on the actions of the source (as in the
above example) assuming that a claim is false on the grounds that the
person defending it is of questionable character
Circular In circular reasoning the author assumes as true it assumes what it seeks to establish
Reasoning what is supposed to be proved
argues circularly by assuming the conclusion is true in
Example: This essay is the best because it is better stating the premises
than all the others.
presupposes the truth of what it sets out to prove
In this example the premise and the conclusion are
identical in meaning. As we know, the conclusion the argument assumes what it is attempting to
should always follow from the premise. In the demonstrate
example above, the premise supports the conclusion,
but the conclusion equally supports the premise, it takes for granted the very claim that it sets out to
creating a circular situation where you can move establish
from premise to conclusion, and then back again to
the premise it offers, in place of support for its conclusion, a mere
restatement of that conclusion
Errors of The authors can either mistake a necessary condition it treats something that is necessary for bringing about a
Conditional for a sufficient condition, or mistake a sufficient state of affairs as something that is sufficient to bring
Reasoning condition for a necessary condition about a state of affairs

confuses a sufficient condition with a required condition


COMMON ERRORS OF REASONING 3
Cause and effect - Assuming a causal relationship on the basis of - mistakes a temporal relationship for a causal
the sequence of events relationship
- Assuming a causal relationship when only a - confusing the coincidence of two events with a causal
correlation exists relation between the two
- Failure to consider an alternate cause for the - assumes a causal relationship where only a
effect, or an alternate cause for both the cause correlation has been indicated
and the effect - fails to exclude an alternative explanation for the
- Failure to consider that the events may be observed effect
reversed - the author mistakes an effect for a cause

Straw man This error occurs when an author attempts to attack - refutes a distorted version of an opposing position
an opponents position by ignoring the actual - incorrectly describing the student representatives
statements made by the opposing speaker and position, thereby making it easier to challenge
instead distorts and refashions the argument, making - portrays opponents views as more extreme than they
it weaker in the process. Often this error is really are
accompanied by the phrase what youre saying is - distorts the proposal advocated by opponents
or if I understand you correctly, which are used to
preface the refashioned and weakened argument
General Lack of Failure to provide any information to support the - The author cites irrelevant data.
Relevant conclusion/ irrelevant information - draws a conclusion that is broader in scope than is
Evidence for the Example: Some critics claim that scientific progress warranted by the evidence advanced
Conclusion has increased the polarization of society and - It uses irrelevant facts to justify a claim about the
alienated large segments of the population. But quality of the disputed product
these critics are wrong because even a cursory - It fails to give any reason for the judgment it
glance at the past shows that society is always reaches.
somewhat polarized and some groups are inevitably - It introduces information unrelated to its conclusion as
alienated. evidence in support of that conclusion
Internal Example: Everyone should join our country club. - bases a conclusion on claims that are inconsistent
contradiction After all, its an exclusive group that links many of with each other
the influential members of the community. - the author makes incompatible assumptions
- introduce information that actually contradicts the
conclusion
- assumes something that it later denies, resulting in a
contradiction
Appeal Fallacies The flaw in this form of reasoning is that the - the judgement of experts is applied to a matter in
Appeal to authority may not have relevant knowledge or all the which their expertise is irrelevant
authority information regarding a situation, or there may a - the argument inappropriately appeals to the authority
difference of opinion among experts as to what is of the mayor
true in the case. Example: use of a neurologist as an - it relies on the judgment of experts in a matter to
COMMON ERRORS OF REASONING 4
Appeal to This error states that a position is true because the - a claim is inferred to be false merely because a
numbers/percent majority believes it to be true majority of people believe it to be false
ages/majority/po - the argument, instead of providing adequate reasons
pular opinion in support of its conclusion, makes an appeal to
popular opinion
Appeal to An Appeal to Emotion occurs when emotions or - attempts to persuade by making an emotional
Emotion emotionally-charged language is used in an attempt appeal
to persuade the reader - uses emotive language in labelling the proposals
Example: Officer, please do not give me a ticket for - the argument appeals to emotion rather than
speeding. In the last month Ive been fired from my reason
job, kicked out of my apartment, and my car broke
down. I dont deserve this!
Survey Errors - Biased sample - uses evidence drawn from a small sample that may
- Improperly constructed/confusing survey well be unrepresentative
questions/ misleading questions (how should the - generalizes from an unrepresentative sample
U.S. government withdraw from the United - states a generalization based on a selection that is
Nations presumes that the U.S. government not representative of the group about which the
should withdraw, a course of action the generalization is supposed to hold true
respondents might not agree with)
- Respondents give inaccurate responses
Exceptional This error takes a small number of instances and - supports a universal claim on the basis of a single
case/overgeneral treats those instances as if they support a broad, example
ization sweeping conclusion - The argument generalizes from too small a sample
of cases
- bases a general claim on a few exceptional
instances
Errors of Composition and division errors involve judgments - assuming that because something is true of each of
Composition and made about groups and parts of a group. An error of the parts of a whole it is true of the whole itself
Division composition occurs when the author attributes a - improperly infers that each and every scientist has a
characteristic of part of the group to the group as a certain characteristic from the premise that most
whole or to each member of the group scientists have that characteristic
Example: Every party I attend is fun and exciting. - takes the view of one lawyer to represent the views
Therefore, my life is fun and exciting of all lawyers
- presumes, without providing justification, that what
An error of division occurs when the author attributes is true of a whole must also be true of its constituent
a characteristic of the whole (or each member of the parts
whole) to a part of the group
Example: The United States is the wealthiest
country in the world. Thus, every American is
COMMON ERRORS OF REASONING 5
Time Shift Errors The mistake involves assuming that conditions will remain constant - treats a claim about what is currently
over time, and that what was the case in the past will be the case in the case as if it were a claim about
the present or future what has been the case for an
Example: The company has always reimbursed me for meals when extended period
Im on a business trip, so they will certainly reimburse me for meals - uncritically draws an inference from
on this business trip. what has been true in the past to
what will be true in the future
Numbers and Many errors in this category are committed when an author - the argument confuses the
Percentage improperly equates a percentage with a definite quantity, or when an percentage of the budget spent on a
Errors author uses quantity information to make a judgment about the program with the overall amount
percentage represented by that quantity spent on that program

Lack of evidence for a position is taken to prove that position is false - treats failure to prove a claim as
Example: The White House has failed to offer any evidence that they constituting denial of that claim
have reached a trade agreement with China. Therefore, no such - taking a lack of evidence for a
agreement has been reached. In the example above the White House claim as evidence undermining that
may have valid reasons for withholding information about the trade claim
agreement. The lack of confirming evidence does not undeniably
prove that a trade agreement has not been reached
Lack of evidence against a position is taken to prove that position is - treating the failure to establish that
true a certain claim is false as equivalent
This error is the opposite of the previous error. Just because no to a demonstration that the claim is
evidence disproving a position has been introduced does not mean true
that the position is true
Example: There has been no evidence given against the existence of
God, so God must exist. The lack of evidence against a position does
not undeniably prove a position
Errors in the Use
of Evidence Some evidence against a position is taken to prove that position is - it confuses undermining an
false argument in support of a given
The introduction of evidence against a position only weakens the conclusion with showing that the
position; it does not necessarily prove the position false. conclusion itself is false
Example: Some historians claim that a lengthy drought preceded the
fall of the Aztec empire. But we know from Aztec writings that in at
least one year during the supposed drought there was minor flooding.
Thus, the claim that there was a lengthy drought prior to the fall of
the Aztec empire is false.
Some evidence for a position is taken to prove that position is true - the argument takes evidence
The introduction of evidence for a position only provides support for showing merely that its conclusion
the position; it does not prove the position to be undeniably true. could be true to constitute evidence
J. PARALLEL REASONING
Question Which one of the following is most closely parallel in its reasoning to the reasoning in the argument above?
Identifiers Which one of the following exhibits a pattern of reasoning most similar to that exhibited by the argument
above?
Which one of the following arguments is most similar in its logical features to the argument above?
Which one of the following arguments is most similar in its pattern of reasoning to the argument above?
The structure of the reasoning in the argument above is most parallel to that in which one of the following?
Primary Objective Identify the answer choice that contains reasoning most similar in structure to the reasoning in the stimulus.
This task requires you to first identify the method of argumentation used by the author and then to match that
reasoning to the reasoning presented in each answer choice

Every Parallel Reasoning stimulus contains an argument and therefore a conclusion. Because your job is to
parallel the argument, you must parallel the subcomponents, including the premises and conclusion

Elements that do Topic of the stimulus be wary of answer choices of the same topic
not have to be
parallel
The order of presentation of the premises and conclusion in the stimulus

1.
1. The
The Method
Method of of Reasoning:
Reasoning: For For example,
example, causal
causal reasoning
reasoning or or conditional
conditional reasoning
reasoning
2.
2. Match
Match the the conclusion
conclusion
-- Match
Match the
the certainty
certainty level
level or
or intent
intent ofof the
the conclusion
conclusion in in the
the stimulus,
stimulus, not
not necessarily
necessarily the
the specific
specific
wording
wording ofof the
the conclusion.
conclusion. ForFor example,
example, aa stimulus
stimulus conclusion
conclusion containing
containing absolutes
absolutes (must,
(must,
never,
never, always)
always) will
will be
be matched
matched by by aa conclusion
conclusion in in the
the correct
correct answer
answer choice
choice using
using similar
similar
absolutes
absolutes
-- DoDo not
not eliminate
eliminate answers
answers just
just because
because the the wording
wording isis not
not identical.
identical. For
For example,
example, majority
majority isis the
the
ELEMENTS THAT same
same asas more
more than
than half
half or
or most
most
HAVE TO BE -- Presence
Presence of of aa negative
negative term
term inin the
the stimulus
stimulus isis not
not grounds
grounds for for dismissing
dismissing the
the answer
answer when
when the
the
PARALLEL stimulus
stimulus has
has positive
positive language.
language. For For example,
example, aa conclusion
conclusion could
could state,
state, The
The councilmember
councilmember must must
be
be present
present at at the
the meeting.
meeting. That
That conclusion
conclusion could
could just
just as
as easily
easily have
have been
been worded
worded as,
as, The
The
councilmember
councilmember must must notnot be
be absent
absent fromfrom the
the meeting
meeting
3.
3. Match
Match the the premise
premise
4.
4. Validity
Validity of of the
the argument:
argument: Always
Always makemake sure
sure to
to eliminate
eliminate anyany answer
answer choice
choice that
that does
does not
not match
match
the
the logical
logical force
force (valid
(valid or
or invalid)
invalid) ofof the
the argument
argument
PARALLEL REASONING - CONTINUED
NUMBERS AND PERCENTAGES CONCEPT
NUMBERS REPRESENT ABSOLUTE VALUES
PERCENTAGES REPRESENT FRACTIONS/PROPORTIONS

Common misconceptions

Increasing percentages Auto manufacturer X increased their United Last Year


automatically leads to increasing States market share from 10% last year to 25% This year
numbers (depends on the size of this year. Therefore, Company X sold more cars Total # of cars sold 1000
the base) in the United States this year than last. 200
Market Share 10%
25%
Xs total car sales 100
50
Decreasing percentages
automatically leads to decreasing
numbers

Increasing numbers automatically The number of bicycle related accidents rose The total number of road accidents could
leads to increasing percentages dramatically from last month to this month. have increased by a greater percentage;
Therefore, bicycle-related accidents must make and hence, it is not necessary that the
up a greater percentage of all road accidents number of bicycle accidents make up a
this month greater percentage of all road accidents
Decreasing numbers automatically
leads to decreasing percentages

Large numbers automatically mean In 2003, Porsche sold just over 18,000 cars in While 18,000 is certainly a large number, it
large percentages, and small the United States. represented only about 1/5 of 20% of total
numbers automatically mean small U.S. car sales in 2003
percentages
Large percentages automatically 90% of 5 vs. 20% of 100
mean large numbers, and small
percentages automatically mean
small numbers
READING
GMAT VERBAL REVIEW
COMPREHENSION
ANALYZING AN RC PASSAGE
WHY is the author ALWAYS
ALWAYS BE
BE MINDFUL
MINDFUL OF
OF THE
THE SCOPE
SCOPE AND
AND TOPIC
TOPIC
writing?
Topic
Topic and
and scope
scope can
can often
often be
be determined
determined in
in the
the first/conclusion
first/conclusion
paragraph
paragraph of
of aa passage
passage
Is it to discuss an
interpretation? SENTENCE
SENTENCE STRUCTURE
STRUCTURE
Critique a study? WATCH
WATCH OUTOUT FOR
FOR TRANSITION
TRANSITION TRIGGERS
TRIGGERS
Transition
Transition triggers
triggers change
change thethe tone
tone oror direction
direction of
of aa
passage.
passage. They often represent a shift in view between two
They often represent a shift in view between
WHO is speaking? experts
experts cited
cited inin aa passage
passage
two

Examples
Examples of of transition
transition triggers
triggers include
include however,
however, but,
but,
although,
although, though, even though, notwithstanding, yet,
though, even though, notwithstanding, yet,
Is the author showing his own despite,
despite, in
in spite
spite of,
of, on
on the
the one
one hand..
hand.. OnOn the
the other
other hand,
hand,
point of view or critiquing an while,
while, unlike
unlike
AA very
very common
common RC RC structure
structure isis for
for one
one expert
expert or
or view
view to
experts point of view? to
be introduced, and then, shortly thereafter,
be introduced, and then, shortly thereafter, a a
contradictory
contradictory expert
expert or or view
view is
is presented
presented

IDEA
IDEA CONTINUITY
CONTINUITY WORDS
WORDS
Similarly,
Similarly, moreover,
moreover, additionally,
additionally, in
in the
the same
same way,
way,
WHAT is the main idea likewise
likewise
or primary purpose? CONCLUSION
CONCLUSION WORDS
WORDS
Thus,
Thus, therefore,
therefore, hence,
hence, so,
so, in
in summary,
summary, in
in conclusion
conclusion
Remember
Remember that
that the
the tone
tone or
or attitude
attitude of
of the
the passage
passage is
is
usually
usually respectful,
respectful, balanced
balanced and
and moderate,
moderate, never
never going
going
to
to extremes
extremes of
of praise
praise nor
nor criticism
criticism

Avoid
Avoid answer
answer choices
choices that
that are
are highly
highly specific
specific and
WHAT is the tone or unequivocal
unequivocal -- always,
always, most,
most, everybody,
everybody, all,
and
all, complete
complete and
and
attitude of the author? never
never

VAGUE,
VAGUE, BROAD
BROAD oror GENERAL
GENERAL answers
answers are
are often
often best
best --
Look for signpost words such as perhaps, may, usually,
Look for signpost words such as perhaps, may, usually,
sometimes, can and some
TYPES OF RC QUESTIONS
"Which of the following best states the central idea of the passage?"
"Which of the following most accurately states the main idea of the passage?" FREQUEN
MAIN IDEA "Which of the following is the principal topic of the passage?" T

"The primary focus of this passage is on which of the following?"


PRIMARY "The main concern of the passage is to..." FREQUEN
"In the passage, the author is primarily interested in...."
PURPOSE "The passage is chiefly concerned with..."
T

TITLE "Which of the following titles best summarizes the passage as a whole?"

SPECIFIC
"According to the passage,...." FREQUEN
DETAIL/TARGE "The passage states that ...." T
T
"It can be inferred that the author makes which of the following assumptions?"
"Which is an assumption underlying the last sentence of the passage?"
"Which of the following, if true, would most strengthen the hypothesis mentioned in lines
INFERENCE 17-19?"
"With which of the following statements regarding chaos theory would the author be
most likely to agree?"

TONE/ATTITUD "The author's attitude towards Morgan's theory could best be described as one of ..."
E
STRUCTURE/ "Which of the following best describes the organization of the passage?"
"Which of the following best describes the organization of the first paragraph of the
ORGANIZATIO passage?"
N "One function of the third paragraph is to...."
TYPES OF RC QUESTIONS STRATEGY
Type of question Strategy
Main idea Look in the first and last paragraphs for the main idea. Any conclusion words like
therefore, thus, so, hence, etc. that you see are most likely introducing the main idea. The
correct answer will say the same thing as it says in the text, but using different words. The
Main Idea is not always stated explicitly in the passage in fact, more likely than not, it is
not stated explicitly. Therefore, in order to answer this type of question when it is more
implicit:

- Re-read the first line of every passage, and the last line of the first and last paragraphs.
This should give you the general structure or outline of the argument, with which you can
answer the Main Idea question
- After determining the general structure or content of the argument, eliminate answer
choices that are too extreme or too broad or too specific, i.e. answer choices that go
beyond the content of the passage, or that deal with content only discussed in
one paragraph of the passage
Primary purpose What is the author trying to do? What is his intention? If he is evaluating a theory, then the
answer could be something like "Discuss an interpretation". Note that the correct answer
would deal with "an interpretation", because the author is only dealing with one theory. If
the Primary Purpose is to criticize 2 new books, then his intention or his primary purpose
might be to critique new studies
- Follow the same strategy as that used in determining the main idea
- NOTE that the overall purpose and main idea can differ from the views of a specific expert
in a passage
Title questions Similar to main idea questions
Specific detail/target - When a fact question directs you to look at a particular line of text for information, you
questions will often find that one of the answer choices is a deceptive one, taken directly from that
line number. More likely than not, there will be something in the sentence or two
before/after the referenced line number that will give you the proper frame for
interpreting the data
- Incorrect answer choices will refer to wrong part of passage, will make sense but not be
mentioned in passage, will be refuted directly in the passage , will stray away from
passages scope and will misinterpret the main point of the author in that section

Inference GMAT inferences go only a tiny bit further than what is said in the passage. When choosing

You might also like